- Joined
- Apr 2025
- Subscription
- Free
Hey! I see your comment is a year old, so I am kind of responding to also see if my thinking is on the right track lol.
the statement "our side will benefit from SHOWING a desire to compromise" is the conclusion. in your comment you are presuming the conclusion to be true as a premise (essentially saying we will benefit from extending the possibility of a compromise, therefore our side will benefit from showing a desire to compromise) we don't know the conclusion to be true (we don't know that showing a desire will actually be beneficial) until we examine the premises
why will we benefit from showing desire to compromise?
because the premises:
either opposition will respond positively to us showing a desire, which will result in a compromise
OR the opposition will respond negatively and they will be held responsible for not reaching a compromise, which benefits us.
so we know in showing a desire to compromise one of these 2 outcomes occur, and the conclusion is regardless of the outcome, showing the desire to compromise is beneficial (meaning both of the outcomes are beneficial, so we must assume that actually reaching a compromise is beneficial)
in sufficient assumption questions we aim to reinforce the support structure from the premises (option 1 or 2) to the conclusion, showing desire is beneficial. if we assume actually reaching a compromise is beneficial, it strengthens one of the premises, which thus strengthens the argument!
#feedback the explanation for #6 is not thorough at all, given so many people get it wrong it would've been helpful to watch an explanation pointing to where in the text A, B, C, and D can be found/inferred.
#help for Q3 'unless' is present, I know we were cautioned that these indicators don't ALWAYS indicate conditional rule (as unless is a G3 indicator) but I am confused about how this particular case wouldn't require us applying the negate sufficient rules (I had initially thought avail > /NA or /4+ weeks > bonded) I can tell this is wrong but I could use an explanation on why this logic is incorrect PLEASE!
#Help !! Am i missing something in the text that implies Caligulas enemies are historians?? and not people of his own era?? when working out C i considered how the answer could imply that his enemies copied earlier works, but it also seemed just as likely that caligula, a cruel tyrant, took inspiration from previous cruel acts of tyants, because i had ASSUMED that his enemies were in his time period (and thus i was like well if his enemies had access to the documentation it is just as likely for Caligula to have access to the documentation). Whereas in the video JY attributes the explanation to historians reading the text and using them in their bias description of Caligula. I can see why C is right given this information about historian enemies, but am at a loss for where that can be found in the text (and how i can avoid this kind of mistake in the future)
#help is there some sort of parameters for what a 'significant number' is?
so there is no concrete reason why #2 B is correct, just that all the other answers are wrong? oh boy.
#help i eliminated D because the author discusses instances where this technique would not be effective, and it appeared D ignored this exception to the rule. I understand why the other answers are incorrect but i struggle to understand why D is correct when it appears to be leaving out a portion of the authors stance towards stealing thunder. does this simply boil down to it being a "most accurately" question, thus the answer doesn't need to be 100%, just closer to accurate than all the other options? thanks!
#feedback This video is very helpful! However the video is 7 minutes long and the lesson says it will only take 1 minute. Additionally I have watched this video 2-3 times and pressed the arrow onto the next lesson but 7sage is still not registering that I have completed this lesson.
EDIT: is my logical error essentially just not realizing the self motivated highly successful people are a subset of highly successful? How do you know when two conditions are tied together "and" and both must occur, versus 2 conditions simultaneously depending on each other? #help
I am confused by this question!!!!
Are being well organized and self motivated are not necessary for being highly successful? A says that if they are not sufficient (highly successful), and they fulfill one of the 1 necessary conditions (organized), they must fail the other (highly successful), is this false reasoning? how is this a reach?
In contrast i struggle to accept C because it hinges on the premise that no self motivated sales person regret their career choice, but don't you need to be self motivated AND organized to be highly successful, so how would C be correct when the premise it is founded on doesn't satisfy the necessary conditions of being successful?
I fear I am all twisted up. This is how i mapped out the stimulus
Well known> Successful > well organized + self motivated (because ALL are BOTH)
/successful /well organized + self motivated
Self motivated > /regret
Is part of the reason E is incorrect because in the passage critics are discussing the indifference of her novels to moral questions, whereas option E is addressing her motivation (which would thus be irrelevant because we are discussing her work). E is unnecessary because even if she wasn't intentionally addressing moral questions, if it is true that her work raised moral questions (for the reasons laid out in the argument, and the assumption in B) then the argument holds true that critics are unfair? Yes? Someone correct my reasoning if I am reaching too far.
Option A is saying that the the reasoning behind a penalty for a crime (i.e jail sentencing, community service hours, fines, etc) is equivalent to what lawmakers think the harm resulting from the crime is (i.e., speeding ticket is not as severe of a punishment because it has little to no harm, whereas m*rder has severe penalty because of the harm). The stimulus claims that harm from bribery and theft are determined to the same, so their penalty is the same.
to put it another way, if it was NOT true that lawmakers mandate penalties for crimes to be proportional to the harm they believe to result from those crimes, it might follow logically for someone to serve life with no parole for spray painting a cop car, but getting 2 hours of community service for spray painting a civilian car (similar harm, vastly different sentencing)
Hope this helps :) sorry if it doesnt :(
i correctly chose B because i recognized the sufficiency necessary confusion by the author, but i hesitated in blind review because I was worried I was inserting personal bias in my answer choice. when i read the stimulus i thought " well we need more than JUST water to develop life, does it have an atmosphere, does it have sunlight?..." #help i find it hard to tell the difference between my inherent bias/assumptions and asking prompting questions related to holes/flaws in arguments. Thank you!
first one i have gotten right in a while! hopefully this means i am getting better at SA, PSA, NA questions as i have found them to extremely difficult thus far
the past three drills have severely impacted my confidence. I was not able to parce out the correct answer in blind review either. #help are there core drills I should be reviewing? I feel I am missing something here.
on both this and the previous question, i have narrowed it down to the correct answer and the answer i chose, but in blind review and timed selection i maintain the wrong answer! i fear i am missing some critical step in comprehension that causes me to eliminate the right answer.
From my understanding, the minor premise contributes to the major premise. Using techniques from previous lectures, IF it is true that exercise apps with home workout solutions are increasing in popularity (minor) is it likely that traditional gyms will anticipate a dip in membership?
Whereas if the roles were reversed, IF it is true that minor premise is traditional gyms can realistically anticipate dips in their memberships is it likely that exercise home workout solutions are gaining increased user engagement? While this logic may seem to work, option 1 is more likely, this requires a large assumption. Perhaps people are no longer using the gym because they have reached their fitness goals, perhaps it is too expensive. The minor premise being gyms loosing membership is not indicative of an increase in online engagement.
This is my best explanation! Good luck to us while studying!
hi there, I am pretty sure the main reason is because ←s→ statements are read reversibly so you cannot take the contrapositive of a ←s→ statement.
A ←s→ B
some A's are B's (at least 1 A is a B)
also means some B's are A's (at least 1 B is an A)